Finding instantaneous rate of change.

Click For Summary
The discussion revolves around finding the instantaneous rate of change (iroc) of the pH level in the mouth after consuming sugar, modeled by the function L(m). The user attempts to apply the limit definition of iroc but encounters confusion with their calculations, particularly in evaluating f(2) and simplifying expressions. They receive feedback about a mistake in their initial evaluation, which leads to further complications in their algebra. The conversation emphasizes the importance of careful arithmetic and algebraic manipulation to progress in solving the problem. Assistance is requested to clarify the next steps in the calculation process.
anonymous12
Messages
29
Reaction score
0

Homework Statement


After you eat something that contains sugar, the pH level in our mouth changes. This can be modeled by the function L(m)=(\frac{-20.4m}{m^2 + 36}) + 6.5 where L is the pH level and m is the number of minutes that have elapsed since eating. Find the instantaneous rate of change in pH level at 2 minutes.


Homework Equations


iroc = instantaneous rate of change
iroc= \lim_{h\to 0} \frac{f(a+h) - f(a)}{h}


The Attempt at a Solution



iroc= \lim_{h\to 0} \frac{f(2+h) - f(2)}{h}
f(2) = 1.02
f(2+h)=(\frac{-20.4(2+h)}{(2+h)^2 + 36}) + 6.5
f(2+h) = \frac{-20.4h-40.8}{h^2 + 4h + 4 + 36} + 6.5
f(2+h) = \frac{-20.4h-40.8}{h^2 + 4h +40} + 6.5
iroc= \lim_{h\to 0} \frac{(\frac{-20.4h-40.8}{h^2 + 4h +40} + 6.5) - 1.02}{h}
I'm really not sure what to do next. I was thinking about subtracting 1.02 from 6.5 which will give me 5.48 and then attempt to add \frac{-20.4h-40.8}{h^2 + 4h +40} + 5.48. If anyone could help me I would really appreciate it.
 
Physics news on Phys.org
When you evaluated f(2) you forgot to add 6.5, which is part of the function.
 
Ok so after I fix the silly mistake, this is how far I get.

iroc= \lim_{h\to 0} \frac{(\frac{-20.4h-40.8}{h^2 + 4h +40} + 6.5) - 5.48}{h}
iroc= \lim_{h\to 0} \frac{(\frac{-1.02^2 - 24.48h-81.6}{h^2 + 4h +40})}{h}

I'm not sure what I should do next.
 
anonymous12 said:
Ok so after I fix the silly mistake, this is how far I get.

iroc= \lim_{h\to 0} \frac{(\frac{-20.4h-40.8}{h^2 + 4h +40} + 6.5) - 5.48}{h}
Notice that, when h=0, you get -1.02+6.5-5.48=0 in the numerator.
iroc= \lim_{h\to 0} \frac{(\frac{-1.02^2 - 24.48h-81.6}{h^2 + 4h +40})}{h}
Notice that, when h=0, you no longer get 0 in the numerator. There is an algebra or arithmetic mistake somewhere.
I'm not sure what I should do next.
If you can fix the algebra, it should become clearer what to do. If not, post back for more help.

p.s. I am moving this problem to the Calculus & Beyond forum.
 
Question: A clock's minute hand has length 4 and its hour hand has length 3. What is the distance between the tips at the moment when it is increasing most rapidly?(Putnam Exam Question) Answer: Making assumption that both the hands moves at constant angular velocities, the answer is ## \sqrt{7} .## But don't you think this assumption is somewhat doubtful and wrong?

Similar threads

  • · Replies 1 ·
Replies
1
Views
4K
  • · Replies 1 ·
Replies
1
Views
16K
  • · Replies 8 ·
Replies
8
Views
2K
Replies
8
Views
2K
  • · Replies 23 ·
Replies
23
Views
3K
  • · Replies 18 ·
Replies
18
Views
2K
  • · Replies 19 ·
Replies
19
Views
9K
  • · Replies 4 ·
Replies
4
Views
2K
  • · Replies 5 ·
Replies
5
Views
4K
  • · Replies 5 ·
Replies
5
Views
7K